LSAT and Law School Admissions Forum

Get expert LSAT preparation and law school admissions advice from PowerScore Test Preparation.

 Administrator
PowerScore Staff
  • PowerScore Staff
  • Posts: 8919
  • Joined: Feb 02, 2011
|
#40161
Complete Question Explanation
(The complete setup for this game can be found here: lsat/viewtopic.php?t=8560)

The correct answer choice is (B)

This question is a direct test of your Not Laws. Since L cannot be at 1 PM (first rule), whereas Z must be at 1 PM (main inference, as explained above), L and Z cannot be given at the same time.

Answer choice (A): This answer choice is incorrect, because L and Y can both be at 2 PM. For instance:
PT73_Game_#2_#9_diagram 1.png
Answer choice (B): This is the correct answer choice. Note that if you did not make this inference ahead of time, this question would require you to “plug-and-chug,” i.e. try out every pair of variables—each at both 1 PM and 2 PM—in order to identify the pair that cannot be at either time slot. This would be an inordinate amount of work, which merely serves to emphasize the importance of investing more time and energy at the initial setup of the game.

Answer choice (C): This answer choice is incorrect, because M and X can both be at 2 PM. For instance:
PT73_Game_#2_#9_diagram 2.png
Answer choice (D): This answer choice is incorrect, because X and Y can both be at 2 PM. For instance:
PT73_Game_#2_#9_diagram 3.png
Answer choice (E): This answer choice is incorrect, because Y and Z can both be at 1 PM. For instance:
PT73_Game_#2_#9_diagram 4.png

Get the most out of your LSAT Prep Plus subscription.

Analyze and track your performance with our Testing and Analytics Package.